Do the primes contain an infinite almost arithmetic progression?












6












$begingroup$


The primes contain finite arithmetic progressions of arbitrary length, but not an infinite arithmetic progression. Say we define an almost arithmetic progression to be a sequence $a_k$, $k geq 0$, such that there exist $a,d$ such that $a_k = a+kd + O(sqrt{k})$. Do the primes contain an infinite almost arithmetic progression?



(The definition is ad hoc and just made up out of curiosity. I wrote the “error term” $O(sqrt{k})$ in analogy to the expectation of a random walk. An obvious generalization of the question is to replace this with some other “small” error term like $O(ln k)$ or whatever.)










share|cite|improve this question









$endgroup$








  • 1




    $begingroup$
    Do you require the terms to be distinct? Also, since the gaps are allowed to grow unboundedly large, in what sense is this kind of sequence similar to an arithmetical progression? If the gaps are allowed to grow, then as Wojowu writes you are really just asking for the asymptotic growth rate of the primes, in a sense.
    $endgroup$
    – Carl Mummert
    Mar 22 at 16:49












  • $begingroup$
    @CarlMummert It was just a poorly thought out question, off the top of my head; I posted it too quickly. I am mulling some similar questions, e.g., about "almost geometric sequences" which might be a little less absurd. I might post that question if it doesn't turn into a pumpkin overnight.
    $endgroup$
    – Zach Teitler
    Mar 23 at 6:50
















6












$begingroup$


The primes contain finite arithmetic progressions of arbitrary length, but not an infinite arithmetic progression. Say we define an almost arithmetic progression to be a sequence $a_k$, $k geq 0$, such that there exist $a,d$ such that $a_k = a+kd + O(sqrt{k})$. Do the primes contain an infinite almost arithmetic progression?



(The definition is ad hoc and just made up out of curiosity. I wrote the “error term” $O(sqrt{k})$ in analogy to the expectation of a random walk. An obvious generalization of the question is to replace this with some other “small” error term like $O(ln k)$ or whatever.)










share|cite|improve this question









$endgroup$








  • 1




    $begingroup$
    Do you require the terms to be distinct? Also, since the gaps are allowed to grow unboundedly large, in what sense is this kind of sequence similar to an arithmetical progression? If the gaps are allowed to grow, then as Wojowu writes you are really just asking for the asymptotic growth rate of the primes, in a sense.
    $endgroup$
    – Carl Mummert
    Mar 22 at 16:49












  • $begingroup$
    @CarlMummert It was just a poorly thought out question, off the top of my head; I posted it too quickly. I am mulling some similar questions, e.g., about "almost geometric sequences" which might be a little less absurd. I might post that question if it doesn't turn into a pumpkin overnight.
    $endgroup$
    – Zach Teitler
    Mar 23 at 6:50














6












6








6


1



$begingroup$


The primes contain finite arithmetic progressions of arbitrary length, but not an infinite arithmetic progression. Say we define an almost arithmetic progression to be a sequence $a_k$, $k geq 0$, such that there exist $a,d$ such that $a_k = a+kd + O(sqrt{k})$. Do the primes contain an infinite almost arithmetic progression?



(The definition is ad hoc and just made up out of curiosity. I wrote the “error term” $O(sqrt{k})$ in analogy to the expectation of a random walk. An obvious generalization of the question is to replace this with some other “small” error term like $O(ln k)$ or whatever.)










share|cite|improve this question









$endgroup$




The primes contain finite arithmetic progressions of arbitrary length, but not an infinite arithmetic progression. Say we define an almost arithmetic progression to be a sequence $a_k$, $k geq 0$, such that there exist $a,d$ such that $a_k = a+kd + O(sqrt{k})$. Do the primes contain an infinite almost arithmetic progression?



(The definition is ad hoc and just made up out of curiosity. I wrote the “error term” $O(sqrt{k})$ in analogy to the expectation of a random walk. An obvious generalization of the question is to replace this with some other “small” error term like $O(ln k)$ or whatever.)







number-theory prime-numbers






share|cite|improve this question













share|cite|improve this question











share|cite|improve this question




share|cite|improve this question










asked Mar 22 at 16:34









Zach TeitlerZach Teitler

2,281419




2,281419








  • 1




    $begingroup$
    Do you require the terms to be distinct? Also, since the gaps are allowed to grow unboundedly large, in what sense is this kind of sequence similar to an arithmetical progression? If the gaps are allowed to grow, then as Wojowu writes you are really just asking for the asymptotic growth rate of the primes, in a sense.
    $endgroup$
    – Carl Mummert
    Mar 22 at 16:49












  • $begingroup$
    @CarlMummert It was just a poorly thought out question, off the top of my head; I posted it too quickly. I am mulling some similar questions, e.g., about "almost geometric sequences" which might be a little less absurd. I might post that question if it doesn't turn into a pumpkin overnight.
    $endgroup$
    – Zach Teitler
    Mar 23 at 6:50














  • 1




    $begingroup$
    Do you require the terms to be distinct? Also, since the gaps are allowed to grow unboundedly large, in what sense is this kind of sequence similar to an arithmetical progression? If the gaps are allowed to grow, then as Wojowu writes you are really just asking for the asymptotic growth rate of the primes, in a sense.
    $endgroup$
    – Carl Mummert
    Mar 22 at 16:49












  • $begingroup$
    @CarlMummert It was just a poorly thought out question, off the top of my head; I posted it too quickly. I am mulling some similar questions, e.g., about "almost geometric sequences" which might be a little less absurd. I might post that question if it doesn't turn into a pumpkin overnight.
    $endgroup$
    – Zach Teitler
    Mar 23 at 6:50








1




1




$begingroup$
Do you require the terms to be distinct? Also, since the gaps are allowed to grow unboundedly large, in what sense is this kind of sequence similar to an arithmetical progression? If the gaps are allowed to grow, then as Wojowu writes you are really just asking for the asymptotic growth rate of the primes, in a sense.
$endgroup$
– Carl Mummert
Mar 22 at 16:49






$begingroup$
Do you require the terms to be distinct? Also, since the gaps are allowed to grow unboundedly large, in what sense is this kind of sequence similar to an arithmetical progression? If the gaps are allowed to grow, then as Wojowu writes you are really just asking for the asymptotic growth rate of the primes, in a sense.
$endgroup$
– Carl Mummert
Mar 22 at 16:49














$begingroup$
@CarlMummert It was just a poorly thought out question, off the top of my head; I posted it too quickly. I am mulling some similar questions, e.g., about "almost geometric sequences" which might be a little less absurd. I might post that question if it doesn't turn into a pumpkin overnight.
$endgroup$
– Zach Teitler
Mar 23 at 6:50




$begingroup$
@CarlMummert It was just a poorly thought out question, off the top of my head; I posted it too quickly. I am mulling some similar questions, e.g., about "almost geometric sequences" which might be a little less absurd. I might post that question if it doesn't turn into a pumpkin overnight.
$endgroup$
– Zach Teitler
Mar 23 at 6:50










3 Answers
3






active

oldest

votes


















11












$begingroup$

Note: I allow for $a_k$ to repeat. If you wish to keep them distinct, the answer by N. S. gives a negative answer.



The answer is: we don't know, but probably. Indeed, your question is equivalent to the question of whether the prime gaps satisfy $p_{n+1}-p_n=O(sqrt{p_n})$, as I explain below. This bound is widely believed to hold - indeed, the running conjecture (Cramér's conjecture) is that we even have $p_{n+1}-p_n=O((log p_n)^2)$, but we are quite far from proving it. The best unconditional bound we have is $p_{n+1}-p_n=O(p_n^{0.525})$, and assuming the Riemann hypothesis we can push this to $O(p_n^{1/2+varepsilon})$, but not to $O(sqrt{p_n})$. (see Wikipedia)



To see the equivalence, take some infinite almost arithmetic progression. Suppose the $O(sqrt{k})$ term is bounded by $Msqrt{k}$. For any prime $p_n$, let $k$ be the least such that $a+dk>p_n+Msqrt{k}$. Then $p_{n+1}leq a_kleq a+dk+Msqrt{k}leq d+p_n+Msqrt{k}+Msqrt{k}=p_n+O(sqrt{k})=p_n+O(sqrt{p_n})$.



Conversely, suppose gaps between primes are $O(sqrt{p_n})$. Take $a=0,d=1$ and $a_k$ the smallest prime greater than $k$. The assumption trivially implies $a_k=k+O(sqrt{k})$.






share|cite|improve this answer









$endgroup$









  • 1




    $begingroup$
    Thanks. So conjecturally $p_n = a + nd + O((log n)^2)$, but the "$a+nd$" part is boring, it can just be $a=0,d=1$ (and not $O(log n)$, that is too strong I take it).
    $endgroup$
    – Zach Teitler
    Mar 22 at 17:20



















11












$begingroup$

The answer is NO.



Note that any set $S$ containing an infinite almost arithmetic progression satisfies
$$
underline{mbox{dens}}(S):= liminf_n frac{(mbox{card}( S cap [0,n]))}{n} geq frac{1}{d}$$



But the primes have density $0$.






share|cite|improve this answer











$endgroup$













  • $begingroup$
    I don't think the question requires $a_k$ to be distinct.
    $endgroup$
    – Wojowu
    Mar 22 at 16:39










  • $begingroup$
    @CarlMummert $a+kd$ has gaps of size $d$. For any $k>d^2$, it is easily possible for two terms to coincide.
    $endgroup$
    – Wojowu
    Mar 22 at 16:47










  • $begingroup$
    Thanks; I was reading it as $O(sqrt{d})$. This makes the question more confusing overall, because if the resulting sequence might not be syndetic, it's not really very similar to an arithmetical progression.
    $endgroup$
    – Carl Mummert
    Mar 22 at 16:48












  • $begingroup$
    @CarlMummert True! I didn't really think it through.
    $endgroup$
    – Zach Teitler
    Mar 22 at 17:17



















4












$begingroup$

The number of primes ≤ N is about N / log (N). Your sequence would have about N / d primes ≤ N in that sequence alone, if N is large. Once N is large enough so that log N >> d, there are just not enough primes around for your sequence.



On the other hand, you will be able to find a very long sequence of primes $a_k$ where



$-10000cdot k^{1/2} ≤ a_k - (3 + 10000k) ≤ 10000 cdot k^{1/2}$.



All you need is $a_0=3$, some prime $3 ≤ a_1 ≤ 20003$, some prime in the range 20003 +/- 14000, one in the range 30003 +/- 17000 etc. This will work until you get to the numbers around $e^{10000}$ where the densities of primes goes too low.






share|cite|improve this answer











$endgroup$













    Your Answer





    StackExchange.ifUsing("editor", function () {
    return StackExchange.using("mathjaxEditing", function () {
    StackExchange.MarkdownEditor.creationCallbacks.add(function (editor, postfix) {
    StackExchange.mathjaxEditing.prepareWmdForMathJax(editor, postfix, [["$", "$"], ["\\(","\\)"]]);
    });
    });
    }, "mathjax-editing");

    StackExchange.ready(function() {
    var channelOptions = {
    tags: "".split(" "),
    id: "69"
    };
    initTagRenderer("".split(" "), "".split(" "), channelOptions);

    StackExchange.using("externalEditor", function() {
    // Have to fire editor after snippets, if snippets enabled
    if (StackExchange.settings.snippets.snippetsEnabled) {
    StackExchange.using("snippets", function() {
    createEditor();
    });
    }
    else {
    createEditor();
    }
    });

    function createEditor() {
    StackExchange.prepareEditor({
    heartbeatType: 'answer',
    autoActivateHeartbeat: false,
    convertImagesToLinks: true,
    noModals: true,
    showLowRepImageUploadWarning: true,
    reputationToPostImages: 10,
    bindNavPrevention: true,
    postfix: "",
    imageUploader: {
    brandingHtml: "Powered by u003ca class="icon-imgur-white" href="https://imgur.com/"u003eu003c/au003e",
    contentPolicyHtml: "User contributions licensed under u003ca href="https://creativecommons.org/licenses/by-sa/3.0/"u003ecc by-sa 3.0 with attribution requiredu003c/au003e u003ca href="https://stackoverflow.com/legal/content-policy"u003e(content policy)u003c/au003e",
    allowUrls: true
    },
    noCode: true, onDemand: true,
    discardSelector: ".discard-answer"
    ,immediatelyShowMarkdownHelp:true
    });


    }
    });














    draft saved

    draft discarded


















    StackExchange.ready(
    function () {
    StackExchange.openid.initPostLogin('.new-post-login', 'https%3a%2f%2fmath.stackexchange.com%2fquestions%2f3158369%2fdo-the-primes-contain-an-infinite-almost-arithmetic-progression%23new-answer', 'question_page');
    }
    );

    Post as a guest















    Required, but never shown

























    3 Answers
    3






    active

    oldest

    votes








    3 Answers
    3






    active

    oldest

    votes









    active

    oldest

    votes






    active

    oldest

    votes









    11












    $begingroup$

    Note: I allow for $a_k$ to repeat. If you wish to keep them distinct, the answer by N. S. gives a negative answer.



    The answer is: we don't know, but probably. Indeed, your question is equivalent to the question of whether the prime gaps satisfy $p_{n+1}-p_n=O(sqrt{p_n})$, as I explain below. This bound is widely believed to hold - indeed, the running conjecture (Cramér's conjecture) is that we even have $p_{n+1}-p_n=O((log p_n)^2)$, but we are quite far from proving it. The best unconditional bound we have is $p_{n+1}-p_n=O(p_n^{0.525})$, and assuming the Riemann hypothesis we can push this to $O(p_n^{1/2+varepsilon})$, but not to $O(sqrt{p_n})$. (see Wikipedia)



    To see the equivalence, take some infinite almost arithmetic progression. Suppose the $O(sqrt{k})$ term is bounded by $Msqrt{k}$. For any prime $p_n$, let $k$ be the least such that $a+dk>p_n+Msqrt{k}$. Then $p_{n+1}leq a_kleq a+dk+Msqrt{k}leq d+p_n+Msqrt{k}+Msqrt{k}=p_n+O(sqrt{k})=p_n+O(sqrt{p_n})$.



    Conversely, suppose gaps between primes are $O(sqrt{p_n})$. Take $a=0,d=1$ and $a_k$ the smallest prime greater than $k$. The assumption trivially implies $a_k=k+O(sqrt{k})$.






    share|cite|improve this answer









    $endgroup$









    • 1




      $begingroup$
      Thanks. So conjecturally $p_n = a + nd + O((log n)^2)$, but the "$a+nd$" part is boring, it can just be $a=0,d=1$ (and not $O(log n)$, that is too strong I take it).
      $endgroup$
      – Zach Teitler
      Mar 22 at 17:20
















    11












    $begingroup$

    Note: I allow for $a_k$ to repeat. If you wish to keep them distinct, the answer by N. S. gives a negative answer.



    The answer is: we don't know, but probably. Indeed, your question is equivalent to the question of whether the prime gaps satisfy $p_{n+1}-p_n=O(sqrt{p_n})$, as I explain below. This bound is widely believed to hold - indeed, the running conjecture (Cramér's conjecture) is that we even have $p_{n+1}-p_n=O((log p_n)^2)$, but we are quite far from proving it. The best unconditional bound we have is $p_{n+1}-p_n=O(p_n^{0.525})$, and assuming the Riemann hypothesis we can push this to $O(p_n^{1/2+varepsilon})$, but not to $O(sqrt{p_n})$. (see Wikipedia)



    To see the equivalence, take some infinite almost arithmetic progression. Suppose the $O(sqrt{k})$ term is bounded by $Msqrt{k}$. For any prime $p_n$, let $k$ be the least such that $a+dk>p_n+Msqrt{k}$. Then $p_{n+1}leq a_kleq a+dk+Msqrt{k}leq d+p_n+Msqrt{k}+Msqrt{k}=p_n+O(sqrt{k})=p_n+O(sqrt{p_n})$.



    Conversely, suppose gaps between primes are $O(sqrt{p_n})$. Take $a=0,d=1$ and $a_k$ the smallest prime greater than $k$. The assumption trivially implies $a_k=k+O(sqrt{k})$.






    share|cite|improve this answer









    $endgroup$









    • 1




      $begingroup$
      Thanks. So conjecturally $p_n = a + nd + O((log n)^2)$, but the "$a+nd$" part is boring, it can just be $a=0,d=1$ (and not $O(log n)$, that is too strong I take it).
      $endgroup$
      – Zach Teitler
      Mar 22 at 17:20














    11












    11








    11





    $begingroup$

    Note: I allow for $a_k$ to repeat. If you wish to keep them distinct, the answer by N. S. gives a negative answer.



    The answer is: we don't know, but probably. Indeed, your question is equivalent to the question of whether the prime gaps satisfy $p_{n+1}-p_n=O(sqrt{p_n})$, as I explain below. This bound is widely believed to hold - indeed, the running conjecture (Cramér's conjecture) is that we even have $p_{n+1}-p_n=O((log p_n)^2)$, but we are quite far from proving it. The best unconditional bound we have is $p_{n+1}-p_n=O(p_n^{0.525})$, and assuming the Riemann hypothesis we can push this to $O(p_n^{1/2+varepsilon})$, but not to $O(sqrt{p_n})$. (see Wikipedia)



    To see the equivalence, take some infinite almost arithmetic progression. Suppose the $O(sqrt{k})$ term is bounded by $Msqrt{k}$. For any prime $p_n$, let $k$ be the least such that $a+dk>p_n+Msqrt{k}$. Then $p_{n+1}leq a_kleq a+dk+Msqrt{k}leq d+p_n+Msqrt{k}+Msqrt{k}=p_n+O(sqrt{k})=p_n+O(sqrt{p_n})$.



    Conversely, suppose gaps between primes are $O(sqrt{p_n})$. Take $a=0,d=1$ and $a_k$ the smallest prime greater than $k$. The assumption trivially implies $a_k=k+O(sqrt{k})$.






    share|cite|improve this answer









    $endgroup$



    Note: I allow for $a_k$ to repeat. If you wish to keep them distinct, the answer by N. S. gives a negative answer.



    The answer is: we don't know, but probably. Indeed, your question is equivalent to the question of whether the prime gaps satisfy $p_{n+1}-p_n=O(sqrt{p_n})$, as I explain below. This bound is widely believed to hold - indeed, the running conjecture (Cramér's conjecture) is that we even have $p_{n+1}-p_n=O((log p_n)^2)$, but we are quite far from proving it. The best unconditional bound we have is $p_{n+1}-p_n=O(p_n^{0.525})$, and assuming the Riemann hypothesis we can push this to $O(p_n^{1/2+varepsilon})$, but not to $O(sqrt{p_n})$. (see Wikipedia)



    To see the equivalence, take some infinite almost arithmetic progression. Suppose the $O(sqrt{k})$ term is bounded by $Msqrt{k}$. For any prime $p_n$, let $k$ be the least such that $a+dk>p_n+Msqrt{k}$. Then $p_{n+1}leq a_kleq a+dk+Msqrt{k}leq d+p_n+Msqrt{k}+Msqrt{k}=p_n+O(sqrt{k})=p_n+O(sqrt{p_n})$.



    Conversely, suppose gaps between primes are $O(sqrt{p_n})$. Take $a=0,d=1$ and $a_k$ the smallest prime greater than $k$. The assumption trivially implies $a_k=k+O(sqrt{k})$.







    share|cite|improve this answer












    share|cite|improve this answer



    share|cite|improve this answer










    answered Mar 22 at 16:52









    WojowuWojowu

    19.2k23174




    19.2k23174








    • 1




      $begingroup$
      Thanks. So conjecturally $p_n = a + nd + O((log n)^2)$, but the "$a+nd$" part is boring, it can just be $a=0,d=1$ (and not $O(log n)$, that is too strong I take it).
      $endgroup$
      – Zach Teitler
      Mar 22 at 17:20














    • 1




      $begingroup$
      Thanks. So conjecturally $p_n = a + nd + O((log n)^2)$, but the "$a+nd$" part is boring, it can just be $a=0,d=1$ (and not $O(log n)$, that is too strong I take it).
      $endgroup$
      – Zach Teitler
      Mar 22 at 17:20








    1




    1




    $begingroup$
    Thanks. So conjecturally $p_n = a + nd + O((log n)^2)$, but the "$a+nd$" part is boring, it can just be $a=0,d=1$ (and not $O(log n)$, that is too strong I take it).
    $endgroup$
    – Zach Teitler
    Mar 22 at 17:20




    $begingroup$
    Thanks. So conjecturally $p_n = a + nd + O((log n)^2)$, but the "$a+nd$" part is boring, it can just be $a=0,d=1$ (and not $O(log n)$, that is too strong I take it).
    $endgroup$
    – Zach Teitler
    Mar 22 at 17:20











    11












    $begingroup$

    The answer is NO.



    Note that any set $S$ containing an infinite almost arithmetic progression satisfies
    $$
    underline{mbox{dens}}(S):= liminf_n frac{(mbox{card}( S cap [0,n]))}{n} geq frac{1}{d}$$



    But the primes have density $0$.






    share|cite|improve this answer











    $endgroup$













    • $begingroup$
      I don't think the question requires $a_k$ to be distinct.
      $endgroup$
      – Wojowu
      Mar 22 at 16:39










    • $begingroup$
      @CarlMummert $a+kd$ has gaps of size $d$. For any $k>d^2$, it is easily possible for two terms to coincide.
      $endgroup$
      – Wojowu
      Mar 22 at 16:47










    • $begingroup$
      Thanks; I was reading it as $O(sqrt{d})$. This makes the question more confusing overall, because if the resulting sequence might not be syndetic, it's not really very similar to an arithmetical progression.
      $endgroup$
      – Carl Mummert
      Mar 22 at 16:48












    • $begingroup$
      @CarlMummert True! I didn't really think it through.
      $endgroup$
      – Zach Teitler
      Mar 22 at 17:17
















    11












    $begingroup$

    The answer is NO.



    Note that any set $S$ containing an infinite almost arithmetic progression satisfies
    $$
    underline{mbox{dens}}(S):= liminf_n frac{(mbox{card}( S cap [0,n]))}{n} geq frac{1}{d}$$



    But the primes have density $0$.






    share|cite|improve this answer











    $endgroup$













    • $begingroup$
      I don't think the question requires $a_k$ to be distinct.
      $endgroup$
      – Wojowu
      Mar 22 at 16:39










    • $begingroup$
      @CarlMummert $a+kd$ has gaps of size $d$. For any $k>d^2$, it is easily possible for two terms to coincide.
      $endgroup$
      – Wojowu
      Mar 22 at 16:47










    • $begingroup$
      Thanks; I was reading it as $O(sqrt{d})$. This makes the question more confusing overall, because if the resulting sequence might not be syndetic, it's not really very similar to an arithmetical progression.
      $endgroup$
      – Carl Mummert
      Mar 22 at 16:48












    • $begingroup$
      @CarlMummert True! I didn't really think it through.
      $endgroup$
      – Zach Teitler
      Mar 22 at 17:17














    11












    11








    11





    $begingroup$

    The answer is NO.



    Note that any set $S$ containing an infinite almost arithmetic progression satisfies
    $$
    underline{mbox{dens}}(S):= liminf_n frac{(mbox{card}( S cap [0,n]))}{n} geq frac{1}{d}$$



    But the primes have density $0$.






    share|cite|improve this answer











    $endgroup$



    The answer is NO.



    Note that any set $S$ containing an infinite almost arithmetic progression satisfies
    $$
    underline{mbox{dens}}(S):= liminf_n frac{(mbox{card}( S cap [0,n]))}{n} geq frac{1}{d}$$



    But the primes have density $0$.







    share|cite|improve this answer














    share|cite|improve this answer



    share|cite|improve this answer








    edited Mar 22 at 16:44

























    answered Mar 22 at 16:39









    N. S.N. S.

    105k7114210




    105k7114210












    • $begingroup$
      I don't think the question requires $a_k$ to be distinct.
      $endgroup$
      – Wojowu
      Mar 22 at 16:39










    • $begingroup$
      @CarlMummert $a+kd$ has gaps of size $d$. For any $k>d^2$, it is easily possible for two terms to coincide.
      $endgroup$
      – Wojowu
      Mar 22 at 16:47










    • $begingroup$
      Thanks; I was reading it as $O(sqrt{d})$. This makes the question more confusing overall, because if the resulting sequence might not be syndetic, it's not really very similar to an arithmetical progression.
      $endgroup$
      – Carl Mummert
      Mar 22 at 16:48












    • $begingroup$
      @CarlMummert True! I didn't really think it through.
      $endgroup$
      – Zach Teitler
      Mar 22 at 17:17


















    • $begingroup$
      I don't think the question requires $a_k$ to be distinct.
      $endgroup$
      – Wojowu
      Mar 22 at 16:39










    • $begingroup$
      @CarlMummert $a+kd$ has gaps of size $d$. For any $k>d^2$, it is easily possible for two terms to coincide.
      $endgroup$
      – Wojowu
      Mar 22 at 16:47










    • $begingroup$
      Thanks; I was reading it as $O(sqrt{d})$. This makes the question more confusing overall, because if the resulting sequence might not be syndetic, it's not really very similar to an arithmetical progression.
      $endgroup$
      – Carl Mummert
      Mar 22 at 16:48












    • $begingroup$
      @CarlMummert True! I didn't really think it through.
      $endgroup$
      – Zach Teitler
      Mar 22 at 17:17
















    $begingroup$
    I don't think the question requires $a_k$ to be distinct.
    $endgroup$
    – Wojowu
    Mar 22 at 16:39




    $begingroup$
    I don't think the question requires $a_k$ to be distinct.
    $endgroup$
    – Wojowu
    Mar 22 at 16:39












    $begingroup$
    @CarlMummert $a+kd$ has gaps of size $d$. For any $k>d^2$, it is easily possible for two terms to coincide.
    $endgroup$
    – Wojowu
    Mar 22 at 16:47




    $begingroup$
    @CarlMummert $a+kd$ has gaps of size $d$. For any $k>d^2$, it is easily possible for two terms to coincide.
    $endgroup$
    – Wojowu
    Mar 22 at 16:47












    $begingroup$
    Thanks; I was reading it as $O(sqrt{d})$. This makes the question more confusing overall, because if the resulting sequence might not be syndetic, it's not really very similar to an arithmetical progression.
    $endgroup$
    – Carl Mummert
    Mar 22 at 16:48






    $begingroup$
    Thanks; I was reading it as $O(sqrt{d})$. This makes the question more confusing overall, because if the resulting sequence might not be syndetic, it's not really very similar to an arithmetical progression.
    $endgroup$
    – Carl Mummert
    Mar 22 at 16:48














    $begingroup$
    @CarlMummert True! I didn't really think it through.
    $endgroup$
    – Zach Teitler
    Mar 22 at 17:17




    $begingroup$
    @CarlMummert True! I didn't really think it through.
    $endgroup$
    – Zach Teitler
    Mar 22 at 17:17











    4












    $begingroup$

    The number of primes ≤ N is about N / log (N). Your sequence would have about N / d primes ≤ N in that sequence alone, if N is large. Once N is large enough so that log N >> d, there are just not enough primes around for your sequence.



    On the other hand, you will be able to find a very long sequence of primes $a_k$ where



    $-10000cdot k^{1/2} ≤ a_k - (3 + 10000k) ≤ 10000 cdot k^{1/2}$.



    All you need is $a_0=3$, some prime $3 ≤ a_1 ≤ 20003$, some prime in the range 20003 +/- 14000, one in the range 30003 +/- 17000 etc. This will work until you get to the numbers around $e^{10000}$ where the densities of primes goes too low.






    share|cite|improve this answer











    $endgroup$


















      4












      $begingroup$

      The number of primes ≤ N is about N / log (N). Your sequence would have about N / d primes ≤ N in that sequence alone, if N is large. Once N is large enough so that log N >> d, there are just not enough primes around for your sequence.



      On the other hand, you will be able to find a very long sequence of primes $a_k$ where



      $-10000cdot k^{1/2} ≤ a_k - (3 + 10000k) ≤ 10000 cdot k^{1/2}$.



      All you need is $a_0=3$, some prime $3 ≤ a_1 ≤ 20003$, some prime in the range 20003 +/- 14000, one in the range 30003 +/- 17000 etc. This will work until you get to the numbers around $e^{10000}$ where the densities of primes goes too low.






      share|cite|improve this answer











      $endgroup$
















        4












        4








        4





        $begingroup$

        The number of primes ≤ N is about N / log (N). Your sequence would have about N / d primes ≤ N in that sequence alone, if N is large. Once N is large enough so that log N >> d, there are just not enough primes around for your sequence.



        On the other hand, you will be able to find a very long sequence of primes $a_k$ where



        $-10000cdot k^{1/2} ≤ a_k - (3 + 10000k) ≤ 10000 cdot k^{1/2}$.



        All you need is $a_0=3$, some prime $3 ≤ a_1 ≤ 20003$, some prime in the range 20003 +/- 14000, one in the range 30003 +/- 17000 etc. This will work until you get to the numbers around $e^{10000}$ where the densities of primes goes too low.






        share|cite|improve this answer











        $endgroup$



        The number of primes ≤ N is about N / log (N). Your sequence would have about N / d primes ≤ N in that sequence alone, if N is large. Once N is large enough so that log N >> d, there are just not enough primes around for your sequence.



        On the other hand, you will be able to find a very long sequence of primes $a_k$ where



        $-10000cdot k^{1/2} ≤ a_k - (3 + 10000k) ≤ 10000 cdot k^{1/2}$.



        All you need is $a_0=3$, some prime $3 ≤ a_1 ≤ 20003$, some prime in the range 20003 +/- 14000, one in the range 30003 +/- 17000 etc. This will work until you get to the numbers around $e^{10000}$ where the densities of primes goes too low.







        share|cite|improve this answer














        share|cite|improve this answer



        share|cite|improve this answer








        edited Mar 22 at 22:05

























        answered Mar 22 at 21:50









        gnasher729gnasher729

        6,1401128




        6,1401128






























            draft saved

            draft discarded




















































            Thanks for contributing an answer to Mathematics Stack Exchange!


            • Please be sure to answer the question. Provide details and share your research!

            But avoid



            • Asking for help, clarification, or responding to other answers.

            • Making statements based on opinion; back them up with references or personal experience.


            Use MathJax to format equations. MathJax reference.


            To learn more, see our tips on writing great answers.




            draft saved


            draft discarded














            StackExchange.ready(
            function () {
            StackExchange.openid.initPostLogin('.new-post-login', 'https%3a%2f%2fmath.stackexchange.com%2fquestions%2f3158369%2fdo-the-primes-contain-an-infinite-almost-arithmetic-progression%23new-answer', 'question_page');
            }
            );

            Post as a guest















            Required, but never shown





















































            Required, but never shown














            Required, but never shown












            Required, but never shown







            Required, but never shown

































            Required, but never shown














            Required, but never shown












            Required, but never shown







            Required, but never shown







            Popular posts from this blog

            Plaza Victoria

            In PowerPoint, is there a keyboard shortcut for bulleted / numbered list?

            How to put 3 figures in Latex with 2 figures side by side and 1 below these side by side images but in...